Must Be True Questions - - Question 12
Governments have only one response to public criticism of socially necessary services: regulation of the activity of ...
Replies
Mehran November 22, 2013
Remember that Must Be True questions are one of the few question on the LSAT that have a check system. The correct answer will be 100% supported by the passage.Here we are presented with an argument. The conclusion is: "the government is certain to respond." WHY? "Governments have only one response to public criticism of socially necessary services: regulation of the activity of providing those services." Also because "public criticism of child-care services has undermined all confidence in such services, and since such services are socially necessary...."
PCSNS ===> RA
not RA ===> not PCSNS
PCSNS = public criticism of socially necessary services
RA = regulation of activity
RA ===> ME
not ME ===> not RA
ME = more expensive
We can combine these statements using the transitive property as follows:
PCSNS ===> RA ===> ME
not ME ===> not RA ===> not PCSNS
(A) is incorrect because it is not 100% supported by the passage. We do not know whether the quality of the socially necessary service will actually improve once the government responds. Quality is never discussed so we cannot infer that just because the government is certain to respond, the quality of child care will improve.
(B) is CORRECT because it is 100% supported by the passage. We know that "governments inevitably make the activity more expensive by regulating it..." Thus, if the government regulates, then it will be more expensive:
PCSNS ===> RA ===> ME
So it must be true that, since the government is certain to respond, the cost of providing child-care services will increase.
(C) is incorrect because it is not 100% supported by the passage. We have not been given any information in the passage on the government using funding to foster advances in child care.
(D) is incorrect because it is not 100% supported by the passage. The passage has told us nothing about what happens when public criticism of policy is strongly voiced.
(E) is incorrect because it is not 100% supported by the passage. As you can see in answer choice (B), we have written out that:
RA ===> ME
not ME ===> not RA
Not being regulated is the necessary condition. Thus, it does not lead us to any conclusion. Remember, Don't Just Reverse! You have to negate and reverse. The existence of the necessary condition does not prove the existence of the sufficient condition. Therefore, based on the information given to us in the passage, we cannot say with 100% certainty that if child-care services are not regulated, the cost of providing child care will not increase.
Hope that was helpful! Please let us know if you have any other questions.
yes90125a2002 September 28, 2017
Why is this an argument and not a set of facts?"Governments have only one response to public criticism of socially necessary services: regulation of the activity of providing those services." = this is a fact
"But governments inevitably make the activity more expensive by regulating it, and that is particularly troublesome in these times of strained financial resources." also seems like a fact